Mobile Robot Control 2024 Ultron:Solution 2: Difference between revisions

From Control Systems Technology Group
Jump to navigation Jump to search
(solution for exercise 2)
Tag: 2017 source edit
 
No edit summary
(One intermediate revision by the same user not shown)
Line 1: Line 1:
Exercise2
Exercise?
 
Hao
 
#In map1 the robot can stop as the designed purpose.[[File:Exercise2-1 Hao.png|thumb]]
#In map2 the robot  stopped when detected the wall on the right side with distance<=0.2[[File:Exercise2-2 Hao.png|thumb]]

Revision as of 12:28, 30 April 2024

Exercise?